Is this set compact in $L^1$

convergence-divergencefunctional-analysismeasure-theoryuniform-integrabilityweak-convergence

Let $(f_n)_{n\in \mathbb{N}}$ be a bounded sequence in $\mathcal{M}(\mathbb{R}^n)\cap L^p(\mathbb{R}^n)$ with $p>1$, where $\mathcal{M}(\mathbb{R}^n)$ denotes the set of all finite measures associated with the $n$ dimensional Lebesgue measure $\lambda$.
Question
Can we find a subsequence which converges weakly in $L^1$?
Due to the bound there is $f\in L^p(\mathbb{R}^n)$ and a subsequence so that along the subsequence we have weak convergence in $L^p$. As $f\in L^p$ we have
$$\forall \varepsilon >0 \exists \delta >0: \forall E \text{ measurable}, \lambda(E)<\delta \Rightarrow \int_{E}|f|^p dx <\frac{\varepsilon}{2}.$$
The same goes now for $f_n$ but dependent on $n$. I would like to use that the last line is also true for $f_n$ uniformly in $n$. Assume for a moment this is true. Then
$$\|f_n\|_{L^1(E)}\leq \|f_n\|_{L^p(E)}\lambda(E)^{\frac{1}{p'}}\leq \varepsilon \delta^{\frac{1}{p'}}.$$
Then by the Dunford-Pettis theorem we conclude that $(f_n)_n$ is uniformly integrable and hence weakly convergent in $L^1$.
Questions to my proof

  • Dunford-Pettis gives the equivalence of uniform integrability and weak convergence in $L^1$. Does weak convergence in $L^p$, $p>1$ already imply uniform integrability in $L^p$?
  • If the latter is true do we only need the boundedness in $\mathcal{M}(\mathbb{R}^n)$ in order to make sure $(f_n)_{n}$ is bounded in $L^1$?

Best Answer

There is no subsequence that converges weakly: Take $f_n = \chi_{[n,n+1]}$. Then $f_n \rightharpoonup 0$ weakly in $L^p(\mathbb R)$ for all $p\in (1,\infty)$ (and weak-star in $L^\infty(\mathbb R) = (L^1(\mathbb R))^*$. But $f_n$ does not converge weakly in $L^1(\mathbb R)$ to zero, since $\int_{\mathbb R} f_n =1 \not\to0$.

If $(f_n)$ would be converging weakly in $L^1$ to some $f$, then we would get $\int_{\mathbb R} f_n \phi \to 0 = \int_{\mathbb R} f \phi$ for all $\phi\in C_c(\mathbb R)$, hence $f=0$. So $(f_n)$ is not weakly converging in $L^1$. (And with the same arguments it has no weakly converging subsequence)

Related Question